Erstellen eines Operators für einen polarisierenden Strahlteiler

Ich versuche, einen polarisierenden Strahlteiler herzustellen, aber alle meine Ansätze sind bisher gescheitert. Ich werde versuchen zu erklären, was ich getan habe, und hoffentlich gibt es da draußen Leute, die auf meine Fehler hinweisen und/oder bessere Methoden vorschlagen können. Ich mache alle Berechnungen in QuTiP, aber jede Hilfe bezüglich der Theorie ist sehr willkommen.

Nehmen wir den Fall mit zwei polarisierten Photonen, die in einen polarisierenden Strahlteiler eingegeben werden. Sie können in den Staaten sein | v (vertikal polarisiert), | H (horizontal polarisiert) oder eine Überlagerung der beiden. | H ist in QuTiP als (2x1)-Vektor definiert, ein sogenanntes "Quantenobjekt": Qobj([[0], [1]])und | v ist Qobj([[1], [0]]). Der Eingangszustand ist das Tensorprodukt der beiden Zustände, wobei die Indizes die Ports bezeichnen, zum Beispiel: | H A | H B .

Die Eingänge befinden sich an den Anschlüssen a und b und die Ausgänge an den Anschlüssen c und d. Damit habe ich diesen einheitlichen PBS-Operator konstruiert:

Ö ^ P B S = [ 0 1 0 0 1 0 0 0 0 0 ich 0 0 0 0 ich ]
Die i sind die Phasen, die von den Reflexionen kommen, und die Zeilen sind folgendermaßen geordnet: ( | H A , | H B , | v A , | v B ) Zu ( | H C , | H D , | v C , | v D ) .

An Port a, kurz vor dem PBS, habe ich eine Halbwellenplatte (HWP) platziert, um die Polarisation eines der Photonen zu drehen. Der Operator sieht so aus:

Ö ^ H W P = [ cos ( 2 θ ) Sünde ( 2 θ ) Sünde ( 2 θ ) cos ( 2 θ ) ]
Wo θ ist der Winkel, mit dem der HWP gedreht wird.

Wenn ich also zwei horizontal polarisierte Photonen eingebe, ist der Zustand nach dem PBS:

| ψ = ( Ö ^ H W P 1 ) Ö ^ P B S | H A | H B
( Ö ^ H W P wirkt nur auf den Zustand im Hafen a)

An Port c und d messe ich die Ausgabe wie folgt:

  1. Ich mache eine Dichtematrix aus dem Ausgabezustand:
    ρ = | ψ ψ |
  2. und ein Messoperator, der beispielsweise nur eine bestimmte Polarisation an beiden Ausgangsports c und d misst | H :

M ^ = | H H |

  1. Den Erwartungswert finde ich per Trace:

M = T R ( M ^ ρ )

In meinen Berechnungen verändere ich die Polarisation des Photons am Port a durch Iteration θ von 0 bis π und dann bewerte ich den Erwartungswert bei jedem Schritt. Für | H Eingang an Port a bzw. b und Messung | H an beiden Ausgangsports bekomme ich:

|H> Eingang an Port a bzw. b und Projektion auf |H> an Port c bzw. d.

Aber das Seltsame passiert, wenn ich etwas eingebe | v an Port a bzw. b und messen | v an beiden Ausgangsports:

|V> Eingabe an Anschluss a bzw. b und Projektion auf |V> an Anschluss c bzw. d.

Hier ist der Erwartungswert konstant.

Also meine Fragen sind:

  1. Was mache ich bei meinen Berechnungen falsch?
  2. Konstruiere ich den PBS-Operator richtig?
  3. Wenn ich nur an Port c messen würde, müsste ich den Zustand an Port d darauf projizieren 1 . Das Problem ist die Tatsache, dass der Messoperator das äußere Produkt von zwei (2x1) Vektoren ist, und ich kann es nicht machen 1 als (2x1)-Vektor. Wie kann ich zum Beispiel einen Messoperator machen, der den Zustand nach dem PBS projiziert | H an Port c und 1 am Hafen d?
  4. Wenn ich einen gut funktionierenden PBS-Betreiber habe, wie trage ich Verluste ein?

Bitte lassen Sie mich wissen, wenn ich etwas näher erläutern muss. Vielen Dank im Voraus!

Wie beschreiben Sie, wenn zwei Photonen zum selben Port gehen? Ihre Codierung scheint Zwei-Photonen-Zustände nicht zuzulassen. Gewöhnlich werden Strahlteiler in zweiter Quantisierung geschrieben (dh Transformieren der Erzeugungsoperatoren für das Feld).
@NorbertSchuch: Haben Sie etwas dagegen, eine Antwort zu geben, in der Sie diesen zweiten Quantisierungsansatz beschreiben, um einen Strahlteiler darzustellen?
@NorbertSchuch Ich vermute nur, aber vielleicht nähern sich die Photonen dem Strahlteiler aus entgegengesetzten Richtungen.
so wie du geschrieben hast | ψ macht nicht viel Sinn: Jeder Staat gefällt | H A Und | H B lebt in einem 4-dimensionalen Raum, ebenso wie die Operatoren Ö P B S Und 1 Ö H W P . Andererseits, | H A | H B ist ein Zustand in einem scheinbar 8 -dimensionaler Raum, wie haben Sie das Produkt dort berechnet?
@NorbertSchuch: danke, aber das scheint für den allgemeinen Fall nicht zu funktionieren.
@flipiefanus Welcher allgemeine Fall? Warum nicht?
@NorbertSchuch: Es scheint nicht die richtigen Ergebnisse für Fock-Zustände zu liefern, da es unverarbeitete Photonen passieren lässt.
@flippiefanus Was sind unverarbeitete Photonen? Ist das wie Rohmilch? Und ja, so kann man einen Strahlteiler quantenmechanisch richtig beschreiben. Das bedeutet, dass es im allgemeinen Fall funktioniert. Wenn es in einigen Fällen nicht funktionieren würde, wäre es nicht richtig.
@NorbertSchuch Dies beschreibt nur zwei Photonen, die von jedem ihrer Ports zum Strahlteiler gehen; Anschluss a und b. Aber ja, wenn man es ist | H und einer ist | v , gehen die Photonen zum selben Ausgangsport. Daher habe ich einen anderen Ansatz ausprobiert, der zwei Photonenzustände beinhaltet. Eine Herausforderung bestand darin, den Vakuumzustand zu definieren. Am Ende habe ich sowohl den Vakuumzustand, ∣H⟩ als auch ∣V⟩, als dreidimensionale Vektoren (noch in QuTiP) definiert, wobei Vakuum Qobj([[1], [0], [0]]), ∣H⟩= ist Qobj([[0], [1], [0]]) und ∣V⟩=Qobj([[0], [0], [1]]). Der Strahlteileroperator wird dann zu einer 81x81-Matrix.
@NorbertSchuch ... Ich habe es immer noch nicht implementiert, bin mir aber immer noch nicht sicher, wie ich Verluste in diesen Operator einbeziehen soll (also Prozentsätze von Reflektivität und Transitivität hinzufügen) und trotzdem einen einheitlichen Operator habe.
@glS | H A Und | H B jeder lebt in einem 2-dim. Platz, also | H A | H B (denken Sie an das Tensorprodukt) lebt in einem 4-Dim. Raum. So auch Ö ^ P B S Und Ö ^ H W P 1 , bzw. Daher ist es möglich, das Produkt so zu berechnen, wie ich es oben in der Beschreibung schreibe.
@ user198150 nein tun sie nicht. Sie benötigen einen zweidimensionalen Raum, um die Polarisation dof anzugeben, und einen anderen, um den Modus anzugeben ( A oder B ), somit gibt es insgesamt vier mögliche Moden und Vektoren wie z | H A müssen in einem 4-dimensionalen Raum leben, um sowohl die Polarisation als auch den Modus zu spezifizieren
@NorbertSchuch Korrektur: die Definitionen sind ∣V⟩=Qobj([[0], [1], [0]]) und ∣H⟩=Qobj([[0], [0], [1]]), as wird in der QuTiP-Dokumentation vorgeschlagen. Ich wollte Erstellungs- und Vernichtungsoperatoren in QuTiP verwenden, um den PBS-Operator zu definieren, aber sie unterscheiden nicht zwischen Polarisierung, wenn sie auf Zustände einwirken, also habe ich auf die oben beschriebene Methode zurückgegriffen.
@glS okay, hier könnte der Kern meines Problems liegen, danke für den Hinweis. In diesem 4-dim. Vektor, sehen diese Definitionen richtig aus: | v A =[1;0;0;0], | H A =[0;1;0;0], | v B =[0;0;1;0], | H B =[0;0;0;1] ?
@ user198150 ja, das sieht richtig aus. Ich habe eine Antwort geschrieben, in der ich das anspreche, was meiner Meinung nach das größte Missverständnis hier war. Ich habe nicht im Detail beschrieben, wie die Formalismen zur Beschreibung der Entwicklung von Viel-Boson-Zuständen funktionieren, da dies langwierig wäre und wahrscheinlich nicht in den Rahmen dieser Frage fallen würde. Wenn Sie auf der Website keine Beschreibungen finden, wie das funktioniert, können Sie sie gerne als separate Frage stellen
@glS Ich bin neu auf dieser Website und habe gerade deine Antwort gesehen. Vielen Dank!! Ja, in meinem Fall habe ich zwei nicht unterscheidbare Photonen als Eingabe für den PBS, also werde ich versuchen, Ihrem Rat zu folgen und zu sehen, ob ich es schaffe, die Berechnung richtig durchzuführen. Wenn ich wieder hängen bleibe, wird wahrscheinlich eine andere Frage auftauchen :)

Antworten (1)

Mit einem Formalismus wie dem, den Sie in Ihrer Frage verwenden, können Sie nach Korrektur einiger Fehler die Entwicklung von Einzelphotonenzuständen korrekt beschreiben .

Insbesondere wird der Zustand eines einzelnen Photons mit zwei räumlichen Freiheitsgraden und zwei möglichen Polarisationszuständen als Vektor in einem vierdimensionalen Raum beschrieben. Eine mögliche Konvention ist zu verwenden

| v A = ( 1 , 0 , 0 , 0 ) T , | H A = ( 0 , 1 , 0 , 0 ) T , | v B = ( 0 , 0 , 1 , 0 ) T , | H B = ( 0 , 0 , 0 , 1 ) T .
Sie können dann die Evolutionsmatrix verwenden, die Sie angegeben haben, um die Evolution jedes Eingangsphotons durch den PBS zu beschreiben.

Dies funktioniert jedoch nicht, sobald Sie mehrere nicht unterscheidbare Photonen als Eingaben haben. Der Grund dafür ist, dass der Raum möglicher Moden vieler ununterscheidbarer Photonen (oder allgemeiner Bosonen) kleiner ist als das Tensorprodukt der Räume der einzelnen Photonen. Grob gesagt liegt das daran, dass wenn die Photonen nicht unterscheidbar sind, Zustände wie | H A | v B Und | v B | H A sind eigentlich der gleiche Zustand.

Um die Entwicklung von Viel-Bosonen-Zuständen richtig zu beschreiben, muss man ihre Ununterscheidbarkeit berücksichtigen. Dies kann auf mehrere äquivalente Arten erfolgen: 1) unter Verwendung des zweiten Quantisierungsformalismus, der automatisch die Symmetrieeigenschaften der Zustände berücksichtigt, 2) unter Verwendung der einheitlichen Evolution, die genau beschreibt, wie sich Basiszustände aus vielen Bosonen entwickeln, oder 3) weiterhin verwenden den Standardformalismus, aber nur die Berechnung der Amplituden zwischen symmetrisierten Eingangs- und Ausgangszuständen.